Question on $sum_{pleq x}f(p)$












0














In the paper of J. Barkley Rosser and Lowell Schoenfeld http://www.seanerikoconnor.freeservers.com/Mathematics/AbstractAlgebra/PrimitivePolynomials/Approximate_Formulas_for_Some_Functions_of_Prime_Numbers.pdf page 68, they obtained the following formula
enter image description here



My question is : should we assume that the integral 2.28 is convergent or this integral is convergent by proof?










share|cite|improve this question






















  • convergence certainly depends on $f$
    – Hagen von Eitzen
    2 days ago
















0














In the paper of J. Barkley Rosser and Lowell Schoenfeld http://www.seanerikoconnor.freeservers.com/Mathematics/AbstractAlgebra/PrimitivePolynomials/Approximate_Formulas_for_Some_Functions_of_Prime_Numbers.pdf page 68, they obtained the following formula
enter image description here



My question is : should we assume that the integral 2.28 is convergent or this integral is convergent by proof?










share|cite|improve this question






















  • convergence certainly depends on $f$
    – Hagen von Eitzen
    2 days ago














0












0








0







In the paper of J. Barkley Rosser and Lowell Schoenfeld http://www.seanerikoconnor.freeservers.com/Mathematics/AbstractAlgebra/PrimitivePolynomials/Approximate_Formulas_for_Some_Functions_of_Prime_Numbers.pdf page 68, they obtained the following formula
enter image description here



My question is : should we assume that the integral 2.28 is convergent or this integral is convergent by proof?










share|cite|improve this question













In the paper of J. Barkley Rosser and Lowell Schoenfeld http://www.seanerikoconnor.freeservers.com/Mathematics/AbstractAlgebra/PrimitivePolynomials/Approximate_Formulas_for_Some_Functions_of_Prime_Numbers.pdf page 68, they obtained the following formula
enter image description here



My question is : should we assume that the integral 2.28 is convergent or this integral is convergent by proof?







number-theory elementary-number-theory






share|cite|improve this question













share|cite|improve this question











share|cite|improve this question




share|cite|improve this question










asked 2 days ago









Theory NombreTheory Nombre

1297




1297












  • convergence certainly depends on $f$
    – Hagen von Eitzen
    2 days ago


















  • convergence certainly depends on $f$
    – Hagen von Eitzen
    2 days ago
















convergence certainly depends on $f$
– Hagen von Eitzen
2 days ago




convergence certainly depends on $f$
– Hagen von Eitzen
2 days ago










1 Answer
1






active

oldest

votes


















0














Your screen-shot cut off half of the explicit answer to your question (emphasis by me):




If the integral in (2.28) below con-



verges, we can rewrite (2.26) as







share|cite|improve this answer





















  • What does means that if the integral not converge? Is the approximation of the sum above false?
    – Theory Nombre
    2 days ago











Your Answer





StackExchange.ifUsing("editor", function () {
return StackExchange.using("mathjaxEditing", function () {
StackExchange.MarkdownEditor.creationCallbacks.add(function (editor, postfix) {
StackExchange.mathjaxEditing.prepareWmdForMathJax(editor, postfix, [["$", "$"], ["\\(","\\)"]]);
});
});
}, "mathjax-editing");

StackExchange.ready(function() {
var channelOptions = {
tags: "".split(" "),
id: "69"
};
initTagRenderer("".split(" "), "".split(" "), channelOptions);

StackExchange.using("externalEditor", function() {
// Have to fire editor after snippets, if snippets enabled
if (StackExchange.settings.snippets.snippetsEnabled) {
StackExchange.using("snippets", function() {
createEditor();
});
}
else {
createEditor();
}
});

function createEditor() {
StackExchange.prepareEditor({
heartbeatType: 'answer',
autoActivateHeartbeat: false,
convertImagesToLinks: true,
noModals: true,
showLowRepImageUploadWarning: true,
reputationToPostImages: 10,
bindNavPrevention: true,
postfix: "",
imageUploader: {
brandingHtml: "Powered by u003ca class="icon-imgur-white" href="https://imgur.com/"u003eu003c/au003e",
contentPolicyHtml: "User contributions licensed under u003ca href="https://creativecommons.org/licenses/by-sa/3.0/"u003ecc by-sa 3.0 with attribution requiredu003c/au003e u003ca href="https://stackoverflow.com/legal/content-policy"u003e(content policy)u003c/au003e",
allowUrls: true
},
noCode: true, onDemand: true,
discardSelector: ".discard-answer"
,immediatelyShowMarkdownHelp:true
});


}
});














draft saved

draft discarded


















StackExchange.ready(
function () {
StackExchange.openid.initPostLogin('.new-post-login', 'https%3a%2f%2fmath.stackexchange.com%2fquestions%2f3062848%2fquestion-on-sum-p-leq-xfp%23new-answer', 'question_page');
}
);

Post as a guest















Required, but never shown

























1 Answer
1






active

oldest

votes








1 Answer
1






active

oldest

votes









active

oldest

votes






active

oldest

votes









0














Your screen-shot cut off half of the explicit answer to your question (emphasis by me):




If the integral in (2.28) below con-



verges, we can rewrite (2.26) as







share|cite|improve this answer





















  • What does means that if the integral not converge? Is the approximation of the sum above false?
    – Theory Nombre
    2 days ago
















0














Your screen-shot cut off half of the explicit answer to your question (emphasis by me):




If the integral in (2.28) below con-



verges, we can rewrite (2.26) as







share|cite|improve this answer





















  • What does means that if the integral not converge? Is the approximation of the sum above false?
    – Theory Nombre
    2 days ago














0












0








0






Your screen-shot cut off half of the explicit answer to your question (emphasis by me):




If the integral in (2.28) below con-



verges, we can rewrite (2.26) as







share|cite|improve this answer












Your screen-shot cut off half of the explicit answer to your question (emphasis by me):




If the integral in (2.28) below con-



verges, we can rewrite (2.26) as








share|cite|improve this answer












share|cite|improve this answer



share|cite|improve this answer










answered 2 days ago









Hagen von EitzenHagen von Eitzen

276k21269496




276k21269496












  • What does means that if the integral not converge? Is the approximation of the sum above false?
    – Theory Nombre
    2 days ago


















  • What does means that if the integral not converge? Is the approximation of the sum above false?
    – Theory Nombre
    2 days ago
















What does means that if the integral not converge? Is the approximation of the sum above false?
– Theory Nombre
2 days ago




What does means that if the integral not converge? Is the approximation of the sum above false?
– Theory Nombre
2 days ago


















draft saved

draft discarded




















































Thanks for contributing an answer to Mathematics Stack Exchange!


  • Please be sure to answer the question. Provide details and share your research!

But avoid



  • Asking for help, clarification, or responding to other answers.

  • Making statements based on opinion; back them up with references or personal experience.


Use MathJax to format equations. MathJax reference.


To learn more, see our tips on writing great answers.





Some of your past answers have not been well-received, and you're in danger of being blocked from answering.


Please pay close attention to the following guidance:


  • Please be sure to answer the question. Provide details and share your research!

But avoid



  • Asking for help, clarification, or responding to other answers.

  • Making statements based on opinion; back them up with references or personal experience.


To learn more, see our tips on writing great answers.




draft saved


draft discarded














StackExchange.ready(
function () {
StackExchange.openid.initPostLogin('.new-post-login', 'https%3a%2f%2fmath.stackexchange.com%2fquestions%2f3062848%2fquestion-on-sum-p-leq-xfp%23new-answer', 'question_page');
}
);

Post as a guest















Required, but never shown





















































Required, but never shown














Required, but never shown












Required, but never shown







Required, but never shown

































Required, but never shown














Required, but never shown












Required, but never shown







Required, but never shown







Popular posts from this blog

Mario Kart Wii

What does “Dominus providebit” mean?

Antonio Litta Visconti Arese